In circle S with � ∠ � � � = 60 m∠RST=60 and � � = 4 RS=4 units find area of sector RST. Round to the nearest hundredth

Answers

Answer 1

If the measure of angle RST is 60°, and RS=4 units, then the area of sector RST is 8.374 square units.

In geometry, a "Sector" of a circle is defined as the portion of circle enclosed by two radii and the arc between them. It can be thought of as a slice or a wedge cut out of a circle.

To find the area of the "sector-RST" in circle centered at "S", we use the formula for area of a sector of a circle, which is :

⇒ Area of sector = (θ/360) × π × r²,

where θ = central angle of sector in degrees, π = 3.14159, and r = radius of circle,

In this case, we are given that m∠RST = 60 degrees and RS = 4 units. Since RS is the radius of the circle centered at "S", we use RS = r,

Substituting the values, θ = 60 degrees, r = RS = 4 units,

We get,

⇒ Area of sector RST = (60/360) × 3.14 × (4)²,

= (1/6) × 3.14 × 16,

= 8.374 square units,

Therefore, the required area of sector is 8.374 square units.

Learn more about Sector here

https://brainly.com/question/29055300

#SPJ4

The given question is incomplete, the complete question is

In circle centered at "S", R and T are the points on the circumference with m∠RST = 60 and RS=4 units .Find area of sector RST.


Related Questions

a plane travels 600 from salt lake city, utah, to oakland, california, with a prevailing wind of 30. the return trip against the wind takes longer. find the average speed of the plane in still air.

Answers

the average speed of the plane in still air is s + 30.

Let's call the average speed of the plane in still air "s" (in miles per hour).

We can use the formula:

time = distance / speed

to find the time it takes the plane to travel from Salt Lake City to Oakland with the wind and against the wind.

With the wind:

time with wind = [tex]600 / (s + 30)[/tex]

Against the wind:

time against wind =[tex]600 / (s - 30)[/tex]

time against wind > time with wind

So we can set up an inequality:

[tex]600 / (s - 30) > 600 / (s + 30)[/tex]

Multiplying both sides by [tex](s - 30)(s + 30)[/tex], we get:

[tex]600(s + 30) > 600(s - 30)[/tex]

Expanding and simplifying, we get:

[tex]600s + 18000 > 600s - 18000[/tex]

Subtracting 600s from both sides, we get:

[tex]18000 > -18000[/tex]

This inequality is true for all values of s. In other words, there are no restrictions on the value of s that would make the return trip take longer than the trip with the wind.

Therefore, we can use the average of the two speeds (with and against the wind) to find the average speed of the plane in still air:

Average speed = [tex]2s(s + 30) / (s + 30 + s - 30)[/tex]

Simplifying, we get:

Average speed = [tex]2s(s + 30) / (2s)[/tex]

Canceling the common factor of 2s, we get:

Average speed = s + 30

We know that the distance from Salt Lake City to Oakland is 600 miles, and we can use the formula:

time = distance / speed

to find the time it takes the plane to travel this distance:

time = [tex]600 / (s + 30)[/tex]

We also know that the return trip (against the wind) takes longer, so we can set up another equation:

time return trip =[tex]600 / (s - 30)[/tex]

We can use these two equations to solve for s:

[tex]600 / (s + 30) = 600 / (s - 30)[/tex]

Cross-multiplying, we get:

[tex]600(s - 30) = 600(s + 30)[/tex]

Expanding and simplifying, we get:

[tex]600s - 18000 = 600s + 18000[/tex]

Subtracting 600s from both sides, we get:

[tex]-18000 = 18000[/tex]

This is not a valid equation, so there must be no solution.

However, we can still find the average speed of the plane in still air by using the equation we derived earlier:

Average speed = s + 30

So the average speed of the plane in still air is s + 30. We don't have a specific value for s, but we can say that the average speed is equal to the speed with the wind plus 30 (which is the speed of the wind).

for such more questions on average speed

https://brainly.com/question/4931057

#SPJ11

Slope-intercept (0, -2) , (9,1)

Answers

Answer:
Do you have a graph or anything?

STRUCTURE The ratio of circumference to diameter is the same for every circle. Is the ratio of circumference to radius the same for every circle? Make sure to explain!

Answers

No, the ratio of circumference to radius is not the same for every circle.

What is ratio?

Ratio refers to the quantitative relation between two or more values, typically expressed in the form of a fraction or a proportion.

According to given information:

No, the ratio of circumference to radius is not the same for every circle. The ratio of circumference to diameter, also known as pi (π), is a constant value that remains the same for every circle. It is approximately equal to 3.14 or 22/7. However, the ratio of circumference to radius varies depending on the size of the circle.

The formula for circumference of a circle is C=2πr, where C is the circumference and r is the radius. Therefore, the ratio of circumference to radius is C/r = 2π. This means that for circles of different sizes, the ratio of circumference to radius will differ since the value of pi remains the same while the radius changes.

For example, if we consider two circles, one with a radius of 2 cm and the other with a radius of 4 cm, the ratio of circumference to radius for the first circle will be 2π (since C = 2πr = 2π x 2 = 4π) and for the second circle, it will be 2π (since C = 2πr = 2π x 4 = 8π). Thus, the ratio of circumference to radius is not the same for every circle.

To know more about ratio visit:

https://brainly.com/question/12024093

#SPJ1

dora drove east at a constant rate of 75 kph. one hour later, tim started driving on the same road at a constant rate of 90 kph. for how long was tim driving, before he caught up to dora? a. 5 hours b. 4 hours c. 3 hours d. 2 hours

Answers

Tim was driving for 5 hours before he caught up to Dora.

The answer is (a) 5 hours.

To solve this problem, we can use the formula:
distance = rate × time
Let's denote the time Tim drove as t hours.

Since Dora started driving one hour earlier, her driving time would be (t + 1) hours.
Dora's distance: 75 kph × (t + 1)
Tim's distance: 90 kph × t
Since Tim catches up to Dora, their distances will be equal:
75(t + 1) = 90t
Now we can solve for t:
75t + 75 = 90t
75 = 15t
t = 5.

The answer is (a) 5 hours.

For similar question on distances.

https://brainly.com/question/29657955

#SPJ11

a large sample of x-y data values are analyzed and reveal a correlation coefficient of-.88. which statement is correct? group of answer choices a weak negative relationship exists. the correlation is weak because r is less than -1. if r had been .88, the correlation would have been much stronger. there is no relation. a fairly strong negative linear relationship exists. *

Answers

The correct statement is that a fairly strong negative linear relationship exists between the x and y variables.

How to find the relationship between the x and y variables of correlation coefficient?

The correlation coefficient is a measure of the strength and direction of the linear relationship between two variables. It ranges from -1 to 1, where a value of -1 indicates a perfect negative correlation, 0 indicates no correlation, and 1 indicates a perfect positive correlation.

In this case, the correlation coefficient is -0.88, which indicates a strong negative linear relationship between the x and y variables. This means that as the value of x increases, the value of y decreases in a predictable manner.

The negative sign of the correlation coefficient indicates that the relationship is negative, meaning that as one variable increases, the other variable tends to decrease. The absolute value of the correlation coefficient, 0.88, indicates a strong relationship, meaning that the values of the two variables are closely related and can be used to predict each other's values.

Therefore, the correct statement is that a fairly strong negative linear relationship exists between the x and y variables.

Learn more about correlation coefficient

brainly.com/question/15577278

#SPJ11

Can someone answer this please and thank you.

Answers

The blue base is the face (put in 1).

The black line is the edge (put in 2).

The dot up top is the vertex (put in 3).

a coin is tossed 10,000 times. what is the chance that the number of heads will be in the range 4850 to 5150?

Answers

The chance that the number of heads will be in the range 4850 to 5150 is approximately 0.9973, or about 99.73%.

The number of heads in 10,000 coin tosses follows a binomial distribution with parameters n = 10,000 (the number of trials) and p = 0.5 (the probability of heads on a single toss).

We can approximate this binomial distribution using the normal distribution, with mean μ = np = 5000 and variance σ² = np(1-p) = 2500.

To find the probability that the number of heads is in the range 4850 to 5150, we can use the normal distribution and standardize the range using the z-score formula:

z = (x - μ) / σ

where x is the number of heads in the range we're interested in.

For the lower bound of 4850, we have:

[tex]z_lower = (4850 - 5000) / \sqrt{(2500)}[/tex]

= -3

For the upper bound of 5150, we have:

[tex]z_upper = (5150 - 5000) / \sqrt{(2500)} = 3[/tex]

Using a standard normal distribution table or calculator, we can find the probability of being within 3 standard deviations of the mean:

P([tex]z_lower[/tex]  < Z < [tex]z_upper[/tex] ) ≈ P(-3 < Z < 3)

= 0.9973.

For similar question on distribution.

https://brainly.com/question/26678388

#SPJ11

The solid below is dilated by a scale factor of 1/2. Find the volume of the
solid created upon dilation.
24
26
10
34

Answers

Answer: 4080

Step-by-step explanation:

First you have to find the area of the triangle. 24*10 = 240. 240/2 = 120. Then you multiply the area of the triangle and multiply it by 34. 120 * 34 = 4080. This means the answer is 4080

Help please? I just need an answer. A clear explanation earns brainliest.

Answers

the simplified  form  of expression is: -(x² + 2x - 2)/((x+2)*(x+4))

what is expression  ?

In mathematics, an expression is a combination of numbers, variables, operators, and/or functions that represents a mathematical quantity or relationship. Expressions can be simple or complex

In the given question,

To evaluate the expression 1/(x+2) - (x+1)/(x+4), we need to find a common denominator for the two terms. The least common multiple of (x+2) and (x+4) is (x+2)(x+4).

So, we can rewrite the expression as:

(1*(x+4) - (x+1)(x+2))/((x+2)(x+4))

Expanding the brackets, we get:

(x+4 - x² - 3x - 2)/((x+2)*(x+4))

Simplifying the numerator, we get:

(-x² - 2x + 2)/((x+2)*(x+4))

Therefore, the simplified expression is:

-(x² + 2x - 2)/((x+2)*(x+4))

To know more about  Expressions , visit:

https://brainly.com/question/14083225

#SPJ1

To determine whether 2126.5
and 58158
are in a proportional relationship, write each ratio as a fraction in simplest form.

What is 2126.5
as a fraction in simplest form?

Enter your answer in the box.

Answers

Answer:

both are 5/13the relationship is proportional

Step-by-step explanation:

You want to know if the fractions (2 1/2)/(6.5) and (5/8)/(1 5/8) are in a proportional relationship, and the simplest form of each.

Fractions

Equivalent fractions can be found by multiplying numerator and denominator by the same number.

  (2 1/2)/(6.5) = 2·(2 1/2)/(2·6.5) = 5/13

  (5/8)/(1 5/8) = 8(5/8)/(8·(1 5/8)) = 5/(8+5) = 5/13

Both fractions are equivalent to 5/13, so their relationship is proportional.

Which expressions are equivalent to 27^4/3?

Select the three correct answers.

A. 4^3

B. (27^1/3)^4

C. 3^1/4

D. 81

Answers

D) 81 is equivalent to 27^(4/3).

The expression 27^4/3 can be simplified using the rule that (a^m)^n = a^(m*n). Therefore, we can write,

27^(4/3) = (3^3)^(4/3)

Using the power of a power rule, we can simplify further,

(3^3)^(4/3) = 3^(3*4/3)

Simplifying the exponent, we get,

3^(4)

To check the other answer choices,

A. 4^3 is not equivalent to 27^4/3.

B. (27^1/3)^4 is equivalent to 27^(4/3), which we already simplified to 3^4. Therefore, this expression is also equivalent to 3^4.

C. 3^1/4 is not equivalent to 27^4/3.

D. 81 is equivalent to 3^(4).

Therefore, the expression 27^4/3 is equivalent to 3^4, which is answer choice D) 81.

To learn more about equivalent here:

https://brainly.com/question/31532746

#SPJ4

PLEASE HELP AND EXPLAIN AND SHOW WORK ON HOW YOU GOT THE ANSWER I WILL MARK YOU BRAINLIEST. PLEASE EXPLAIN HOW YOU GOT THE ANSWER!!!

Answers

The terms arranged in order from smallest to biggest are: (-2)³, -√25, √11, 10, and 4² after comparing the values of the final numbers.

How to arrange the terms of numbers in ascending order

We shall first simplify the numbers to get their final values and then compare to which is smaller as follows:

4² = 4 × 4 = 16

-√25 = -5

10 = 10

√11 = 3.3166

(-2)³ = -2 × -2 × -2 = -8

In conclusion, we have by comparing the final values of the numbers the terms arranged from smallest to the biggest as: (-2)³, -√25, √11, 10, and 4².

Read more about numbers here:https://brainly.com/question/1094377

#SPJ1

I need help please I will give brainliest to the best answer...

Answers

The value of x in the intersecting chords that extend outside circle is 5

Calculating the value of x

From the question, we have the following parameters that can be used in our computation:

intersecting chords that extend outside circle

Using the theorem of intersecting chords, we have

4 * (x + 6 + 4) = 6 * (x - 1 + 6)

Evaluate the like terms

So, we have

4 * (x + 10) = 6 * (x + 5)

Using a graphing tool, we have

x = 5

Hence. the value of x is 5

Read more about intersecting chords at

https://brainly.com/question/13950364

#SPJ1

what is 72% written in a deciamal

Answers

the answer is 0.72….

40000 is divided by the smallest number so that the result is a perfect cube. find the cube root of the resulting number.

Answers

The Cube root of the resulting number is 8.

The smallest number that 40000 can be divided by so that the result is a perfect cube, we need to factorize 40000 into its prime factors:

[tex]40000 = 2^6 \times 5^4[/tex]

To make this a perfect cube, we need to ensure that the powers of each prime factor are multiples of 3.

The smallest number we can divide 40000 by so that the result is a perfect cube is:

[tex]40000 = 2^6 \times 5^4[/tex]

Now we can find the cube root of the resulting number:

[tex]3\sqrt (40000 \div 100) = 3\sqrt400 = 8.[/tex]

Factories 40000 into its prime components in order to determine.

The least number that the result may be divided by while still producing a perfect cube.

The powers of each prime factor must be multiples of three in order for this to be a perfect cube.

The least number that 40000 may be divided by to produce a perfect cube is:

For similar questions on Cube Root

https://brainly.com/question/26726803

#SPJ11

erin is playing darts at the adventure arcade. she scores a bullseye 15% of the time, and she is about to throw 5 darts. how likely is it that she will get at least one bullseye?

Answers

the likelihood of Erin getting at least one bullseye in 5 throws is 0.5563 or 55.63%.

To calculate the likelihood of Erin getting at least one bullseye, we need to first calculate the probability of her not getting a bullseye in a single throw. Since she scores a bullseye 15% of the time, the probability of her not getting a bullseye in a single throw is 85% (100% - 15%).

Using the probability of not getting a bullseye in a single throw, we can use the following formula to calculate the probability of not getting a bullseye in all 5 throws:

0.85 x 0.85 x 0.85 x 0.85 x 0.85 = 0.4437

Therefore, the probability of Erin not getting a bullseye in all 5 throws is 0.4437 or 44.37%.

To calculate the probability of Erin getting at least one bullseye in 5 throws, we can subtract the probability of her not getting a bullseye in all 5 throws from 1:

1 - 0.4437 = 0.5563

Therefore, the likelihood of Erin getting at least one bullseye in 5 throws is 0.5563 or 55.63%.

learn more about probability

https://brainly.com/question/30034780

#SPJ11

The probability that Erin will get at least one bullseye in her 5 throws at the adventure arcade is approximately 55.63%.

To find the probability that she will get at least one bullseye in 5 throws, we can use the complementary probability.

This means we will first find the probability of her not getting a bullseye in all 5 throws, and then subtract that from 1.

Find the probability of not getting a bullseye (1 - bullseye probability)

1 - 0.15 = 0.85

Calculate the probability of not getting a bullseye in all 5 throws

0.85^5 ≈ 0.4437

Find the complementary probability (probability of at least one bullseye)

1 - 0.4437 ≈ 0.5563

So, the probability that Erin will get at least one bullseye in her 5 throws at the adventure arcade is approximately 55.63%.

for such more question on probability

https://brainly.com/question/13604758

#SPJ11

Round the number. Write the result as the product of a single digit and a power of 10.
4,241,933,200

Answers

Rounding 4,241,933,200 to a single digit times a power of 10 would result in:

4.2 × 10^9

To round to this value, we drop all the digits after the tens digit (which is 2), and then round the tens digit up to 3 because the digit to its right (which is 9) is greater than or equal to 5. Finally, we append nine zeros to the end of the number to represent the power of 10.

Solve for x to make A||B.
A = x + 12
B = x + 48

X = [?]

Answers

Answer:

Step-by-step explanation:= x+48=180 ( linier pair )

                                            = x=180-48

                                            = x=132

                                         

                                              = x+12=180 (liner pair)

                                              = x=180-12

                                              = x=168

Compare the numbers using <, >, or =. 0. 78 ___ 0. 708 < > =

Answers

For the given numbers, 78 < 0. 708

To compare two numbers, we need to look at their values and determine which one is larger or smaller. In this case, we have 78 and 0.708. We can start by comparing their whole number parts, which are 78 and 0, respectively. Since 78 is greater than 0, we know that 78 is a larger number.

But what about the decimal parts of these numbers? To compare them, we need to look at the place value of each digit. The first digit after the decimal point in 78 is 0, and the first digit after the decimal point in 0.708 is 7. Since 7 is greater than 0, we know that 0.708 is a larger number than 0.78 in terms of their decimal parts.

Now that we have compared the whole number parts and decimal parts separately, we can combine the results to determine the final comparison. Since 78 is larger than 0 and 0.708 is larger than 0.78 in terms of their decimal parts, we can conclude that:

78 < 0.708

We use the symbol "<" here because 78 is smaller than 0.708.

To know more about number here

https://brainly.com/question/17429689

#SPJ4

Eddie Clauer sells a wide variety of outdoor equipment and clothing. The company sells both through mail order and via the internet. Random samples of sales receipts were studied for mail-order sales and internet sales, with the total purchase being recorded for each sale. A random sample of 17 sales receipts for mail-order sales results in a mean sale amount of $84. 80 with a standard deviation of $19. 25. A random sample of 12 sales receipts for internet sales results in a mean sale amount of $77. 10 with a standard deviation of $26. 25. Using this data, find the 90% confidence interval for the true mean difference between the mean amount of mail-order purchases and the mean amount of internet purchases. Assume that the population variances are not equal and that the two populations are normally distributed.

Step 1 of 3 :

Find the critical value that should be used in constructing the confidence interval. Round your answer to three decimal places.

Step 2 of 3

Find the Staandard error of the sampling distrbution to be used in constructing the confidence interval

Step 3 of 3

you were to ask to construct the 90% confidence interval, given the following information

Answers

The 90% confidence interval for the true mean difference between the mean amount of mail-order purchases and the mean amount of internet purchases is approximately [-6.62, 22.02].

The critical value that should be used in constructing the confidence interval.

Since we are looking for a 90% confidence interval, we need to find the critical value associated with a 5% level of significance in a two-tailed test.

Using a t-distribution with (n1-1) + (n2-1) degrees of freedom and a significance level of 0.05, we find the critical value to be:

t-critical = 1.717 (using a t-distribution table or a calculator)

Step 2 of 3:

Next, we need to find the standard error of the sampling distribution to be used in constructing the confidence interval.

Since the population variances are not equal, we need to use the Welch-Satterthwaite equation to calculate the standard error:

SE = sqrt[([tex]s1^2[/tex]/n1) + ([tex]s2^2[/tex]/n2)]

where s1 and s2 are the sample standard deviations, and n1 and n2 are the sample sizes.

Substituting the given values, we get:

SE = sqrt[([tex]19.25^2[/tex]/17) + ([tex]26.25^2[/tex]/12)]

SE ≈ 8.35

Step 3 of 3:

To construct the 90% confidence interval, we can use the formula:

(mean1 - mean2) ± t-critical * SE

where mean1 and mean2 are the sample means, and t-critical and SE are the values calculated in steps 1 and 2.

Substituting the given values, we get:

= (84.80 - 77.10) ± 1.717 x 8.35

= 7.70 ± 14.32

Therefore,

The 90% confidence interval for the true mean difference between the mean amount of mail-order purchases and the mean amount of internet purchases is (approx) [-6.62, 22.02].

We can be 90% confident that the true mean difference between the mean amount of mail-order purchases and the mean amount of internet purchases falls within this interval.

For similar question on confidence interval:

https://brainly.com/question/20309162

#SPJ11

Please help.
If the radius of the clock is 24 cm and the distance from the top of the clock at point D to the hanger at point B is 2 cm, what is the length from point A to point B?

2 cm
10 cm
12 cm
24 cm

Answers

The length from point A to point B on the clock is approximately 24.083 cm, which is closest to 24 cm. This is calculated using the Pythagorean theorem.

Using the Pythagorean theorem, we can calculate the length from point A to point B as follows

First, we need to find the length of the vertical line segment from point D to point A. This is equal to the radius of the clock, which is 24 cm.

Next, we can find the length of the horizontal line segment from point D to point B. This is equal to the distance from the top of the clock at point D to the hanger at point B, which is given as 2 cm.

Now, we can use the Pythagorean theorem to find the length from point A to point B

AB² = AD² + DB²

AB² = (24 cm)² + (2 cm)²

AB² = 576 cm² + 4 cm²

AB² = 580 cm²

AB ≈ 24.083 cm

Therefore, the length from point A to point B is approximately 24.083 cm, which is closest to 24 cm.

To know more about Pythagorean theorem:

https://brainly.com/question/14930619

#SPJ1

Answer:

The length from point A to point B on the clock is approximately 24.083 cm, which is closest to 24 cm. This is calculated using the Pythagorean theorem.

Hope this helps :)

Pls brainliest...

please solve correctly my grade depends on it

Answers

Just use the pythagorean theorem to solve the hypotenuse!

(3^2)+(2^2)=x^2

9+4=13^2

[tex]\sqrt{13}[/tex] = [tex]\sqrt{x}[/tex]

[tex]13^{2}[/tex] km

Hope this helps <3

Find the points on the surface z2 = xy +16 closest to the origin. The points on the surface closest to the origin are (Type an ordered triple. Use a comma to separate answers as needed. )

Answers

The points on the surface z² = xy + 16 closest to the origin are: (-4,4,0) and (4, -4, 0)

We know that the distance between an arbitrary point on the surface and the origin is d(x, y, z) = √(x² + y² + z²)

Using Lagrange multipliers,

L(x, y, z, λ) = x² + y² + z² + λ(z² - xy - 16)

We have partial derivatives.

[tex]L_x[/tex] = 2x - λy

[tex]L_y[/tex] = 2y - λx

[tex]L_z[/tex] = 2z + 2zλ

[tex]L_\lambda[/tex] = z² - xy - 16

Now we set each partial derivative to zero to find critical points.

[tex]L_x[/tex] = 0

2x - λy = 0

[tex]L_y[/tex] = 0

2y - λx = 0

After solving above equations simultaneously we get (x + y)(x - y) = 0

i.e., x = -y   OR   x = y

[tex]L_z[/tex] = 0

2z + 2zλ = 0

z = 0  OR  λ = 0

Consider [tex]L_\lambda[/tex] = 0

z² - xy - 16 = 0

-xy = 16                  ............(as z = 0)

when x = y then -y² = 16 which is not true.

So, consider x = -y

-(-y)y = 16

y² = 16

y = ±4

when y = 4 then we get x = -4

and when y = -4 then we get x = 4

Therefore, the closest points are:(-4,4,0) and (4, -4, 0)

Learn more about the Lagrange multipliers here:

https://brainly.com/question/30776684

#SPJ4

Using the graph, determine the coordinates of the x-intercepts of the parabola.

Answers

Answer:

x = -5, x = 1

As (x, y) coordinates, the x-intercepts are (-5, 0) and (1, 0).

Step-by-step explanation:

The x-intercepts are the x-values of the points at which the curve crosses the x-axis, so when y = 0.

From inspection of the given graph, we can see that the parabola crosses the x-axis at x = -5 and x = 1.

Therefore, the x-intercepts of the parabola are:

x = -5x = 1

As (x, y) coordinates, the x-intercepts are (-5, 0) and (1, 0).

the x intercepts are 1 and-5

Red=10
blue=8
yellow=5
what is the ratio of red balls to blue balls?

Answers

Answer:1.25

Step-by-step explanation:

it just math

Someone help me i will give brainliest!

Answers

The probability that the golfer will hit at least 6 times in his next 10 attempts is A. 20 %

How to find the probability ?

To estimate the probability of the golfer hitting at least 6 times in his next 10 attempts using a table of random numbers, we can perform a simulation.

Let's use the given table of random numbers to simulate 10 attempts for each trial. We can consider each pair of digits as one attempt. We will perform 10 trials and count how many times the golfer hits at least 6 times in 10 attempts.

Now count the number of trials with at least 6 hits:

Trial 2, Trial 5, and Trial 9 have at least 6 hits. That's 3 out of 10 trials.

To estimate the probability, divide the number of successful trials (at least 6 hits) by the total number of trials:

Probability = (Number of successful trials) / (Total number of trials)

Probability = 3 / 10 = 0.3

The estimated probability that the golfer will hit at least 6 times in his next 10 attempts is 30%. There is no exact match among the possible answers, but the closest one is 20%.

Find out more on probability at https://brainly.com/question/30618573

#SPJ1

Determine whether ▰ABCD with vertices A(-4,6), B(-1,7), C(0,4), and D(-3,3) is a rhombus, a rectangle, a square, or none. Select all the apply.

~a.) Rhombus
~b.) Rectangle
~c.) Square
~d.) None

Answers

The only statement that is true is b, which states that the quadrilateral is a rectangle.

What is quadrilateral?

A quadrilateral is a polygon with four sides and four vertices. The sum of the interior angles of a quadrilateral is always 360 degrees. Quadrilaterals can have sides of different lengths and angles of different measures, giving rise to many different types of quadrilaterals with different properties.

According to the given information

First, we find the lengths of the sides of the quadrilateral:

AB = √[(7-6)² + (-1+4)²] = √10

BC = √[(4-7)² + (0-0)²] = 3

CD = √[(3-4)² + (-3+0)²] = √10

AD = √[(6-3)² + (-4+1)²] = √26

Then, we find the slopes of each pair of opposite sides:

AB: (7-6)/(−1+4) = 1/3

BC: (4-0)/(0-(-1)) = 4/1 = 4

CD: (-3-(-4))/(0-(-3)) = 1/3

AD: (6-3)/(-4-(-1)) = -1/5

Now we can analyze each statement:

a.) Rhombus

A rhombus is a quadrilateral with all sides of equal length. We found that AB = CD and AD ≠ BC, so not all sides are of equal length. Therefore, statement a is false.

b.) Rectangle

A rectangle is a quadrilateral with all angles equal to 90 degrees. We can find the slopes of adjacent sides and check if they are opposite reciprocals:

AB: 1/3

BC: 4

CD: 1/3

AD: -1/5

We can see that AB and CD have slopes of 1/3 and are opposite reciprocals, and BC and AD have slopes of 4 and -1/5, respectively, and are also opposite reciprocals. Therefore, all angles of the quadrilateral are 90 degrees. Also, since AB = CD and AD ≠ BC, the quadrilateral is a rectangle. Therefore, statement b is true.

c.) Square

A square is a special type of rectangle with all sides of equal length. We found that AB ≠ AD, so not all sides are of equal length. Therefore, statement c is false.

d.) None

We have determined that the quadrilateral is a rectangle, so it is not "none". Therefore, statement d is false.

Therefore, the only statement that is true is b, which states that the quadrilateral is a rectangle.

To know more about the quadrilateral and rectangle visit:

brainly.com/question/11936810

#SPJ1

a p-value a. can be positive or negative. b. is a probability. c. can be smaller than 0 but no larger than 1. d. can be larger than 1 but no smaller than 0. e. can only range in value from -1 to 1.

Answers

A p-value is a probability.

A p-value is the probability of obtaining a test statistic as extreme or more extreme.

The observed value, assuming the null hypothesis is true.

It ranges in value from 0 to 1 and represents the strength of evidence against the null hypothesis.

A p-value cannot be negative, as it is a probability and probabilities are always between 0 and 1.

A p-value also cannot be larger than 1, as it represents a probability.

A probability cannot exceed 1.

Finally, a p-value cannot be smaller than 0, as it represents a probability.

A probability cannot be negative.

the correct option is b. is a probability.

For similar questions on P-Value

https://brainly.com/question/13786078

#SPJ11

slove and answer x+y=11 2x-y=19

Answers

Answer:

x + y = 11

2x - y = 19

--------------

3x = 30

x = 10, so y = 1

Write the functions in standard form:
h(x)=2(x-3)²-9
h(x)=
p(x) = -5(x + 2)² + 15
p(x)=

Answers

Answer:

[tex]h(x)=2x^2-12x+9[/tex],  [tex]p(x)=-5x^2-20x-5[/tex]

Step-by-step explanation:

To get to the standard form of a quadratic equation, we need to expand and simplify. Recall that standard form is written like so:

[tex]ax^2+bx+c[/tex]

Where a, b, and c are constants.

Let's expand and simplify h(x).

[tex]2(x-3)^2-9=\\2(x^2+9-6x)-9=\\2x^2+18-12x-9=\\2x^2+9-12x=\\2x^2-12x+9[/tex]

Thus, [tex]h(x)=2x^2-12x+9[/tex]

Let's do the same for p(x).

[tex]-5(x+2)^2+15=\\-5(x^2+4+4x)+15=\\-5x^2-20-20x+15=\\-5x^2-5-20x=\\-5x^2-20x-5[/tex]

Thus, [tex]p(x)=-5x^2-20x-5[/tex]

Other Questions
Use the image to determine the direction and angle of rotation. 270 clockwise rotation 270 counterclockwise rotation 90 clockwise rotation 180 clockwise rotation There is no valid reason for ever delegating a task that the supervisor can perform better and faste (true or false) felippe, a 17-year-old high school senior, is very talkative, intelligent, assertive, and politically conservative. research suggests that he would be most likely to develop a close friendship with esther, a manager at a customer service call center, reprimands her subordinates each time they are late to work. thus, esther is using the bioavailability of which two vitamins is significantly higher in supplemental form as compared to what is naturally occurring in foods? _________________________ an awareness among medical students that the knowledge base of medicine is incomplete. Here are two complex numbers being multiplied: (4 + 2)(6-3) = ?Without calculating the exact result of the multiplication, how canyou tell that the result will be a real number? consider a reaction between two gaseous reactants (4 mol of a and 4 mol of b) in the closed flasks shown below. assume that the two reactions are both at room temperature. which reaction will occur faster? You are considering an investment in a AAA-rated U.S. corporate bond but you are not sure what rate of interest it should pay. Assume that the real risk-free rate of interest is 1.0%; inflation is expected to be 1.5%; the maturity risk premium is 2.5%; and, the default risk premium for AAA-rated corporate bonds is 3.5%. What rate of interest should the U.S. corporate bond pay?a. 5.0%b. 8.5%c. 2.5%d. 6.0% Rectangle MPAT has vertices M(1,2) , P(1, 3), A(3, 3), and T(3, 2) . Rectangle MPAT . Which coordinates describe the vertices of the image? 21. which part of the prefrontal cortex seems to be particularly involved in working memory functions? 4- Calculate the pH of 0.3 M NH, where is K = 1.7 x 10 Find the differential of the function. Z = e4x cos(4t) Problem 7: Find the surface area and round to the nearest tenth. one of the techniques used in this experiment was that of crystallization. when cooling a solution in the process of crystallization, why would an ice bath be preferable over cold water or ice alone? none of the answers shown are correct. ice is too cold and will freeze any solution. cold water would dilute the solution making it impossible for crystals to form. a mixture of ice and water will keep the temperature above freezing and will contact the entire portion of the container immersed in the ice/water mixture. a client who has been discharged home on citalopram calls the nurse reporting that the medication causes the client to feel too drowsy. the nurse should make which suggestion? 4 (20) + 6 (45) - (80) how are the masses of supermassive black holes related to the masses of the bulges of their surrounding galaxies and what does this suggest about the role of supermassive black holes in galaxy evolution. 7 Skloped Quad Enterprises is considering a new 3-year expansion project that requires an initial fixed asset investment of $5.184 million. The fixed asset will be depreciated straight-line to zero over its 3-year tax life, after which time it will have a market value of $403,200. The project requires an initial investment in net working capital of $576,000. The project is estimated to generate $4,608,000 in annual sales, with costs of $1,843,200. The tax rate is 23 percent and the required return on the project is 11 percent What is the project's Year Onet cash flow? Year 0 cash flow eBook References What is the project's Year 1 net cash flow? Year 1 cash flow What is the project's Year 2 net cash flow? Year 2 cash flow What is the project's Year 3 net cash flow? Year 3 cash flow What is the NPV? NPV you want to rent an unfurnished one-bedroom apartment for next semester. the mean monthy rent for a random sample of 45 apartments advertised in the local newspaper is $775. assume the standard deviation is $110. find a formula used to generate the 98% confidence interval for the mean monthly rent for unfurnished one-bedroom apartments available for rent in this community.